Apa Hubungan antara QMA dan AM?

12

Saya baca di SP Yordania, D. Gosset, "PJ Cinta masalah -Lengkap untuk Hamiltonians stoquastic dan matriks MarkovQMA " bahwa tidak mungkin bahwa .QMAAM

Saya terkejut dengan pernyataan ini. Jadi apa hubungan yang tepat antara dan A M ?QMAAM

Zelah 02
sumber
@ Kaveh, edit judul Anda salah. Kata "stoquastic" dieja dengan benar. Kebingungan yang sama terjadi dalam komentar cstheory.stackexchange.com/questions/3161/…
Alessandro Cosentino
1
@Alessandro Cosentino: Saya mengubahnya kembali menjadi stoquastic, terima kasih.
Kaveh

Jawaban:

22

Tidak ada hubungan yang diketahui antara QMA dan AM, dan masuk akal untuk menduga mereka tidak ada bandingannya.

Jika QMA terbukti terkandung dalam AM, itu akan menjadi hasil yang sangat besar dalam kompleksitas kuantum. Tentu saja itu akan menyiratkan bahwa BQP dalam PH, yang itu sendiri akan sangat besar, tetapi akan melampaui itu - itu pasti akan membutuhkan wahyu besar tentang struktur algoritma kuantum dan sertifikat kuantum.

Karena itu, bukti terhadap tidak terlalu meyakinkan. Peramal yang QMA tidak terkandung dalam AM akan membantu, dan sepertinya hasil seperti itu mungkin tidak jauh - tapi kami bahkan belum memilikinya.

Sebuah bukti penahanan terbalik, AM di QMA, juga akan sangat besar. Setidaknya di sini kita memiliki nubuat relatif yang AM tidak terkandung dalam QMA (dan bahkan tidak terkandung dalam PP).

John Watrous
sumber
Apakah BQP terkandung dalam QMA? Saya bertanya karena padanan "klasik" (BPP vs NP) tidak diketahui sama sekali. (ini dari bacaan saya tentang komentar Anda "itu akan menyiratkan bahwa BQP ada di PH"
Suresh Venkat
5
@ Suresh: Ya, benar. BQP dan QMA memiliki hubungan yang sama dengan P dan NP, atau BPP dan MA. Dalam tiga contoh ini, kelas pertama sepele dalam yang kedua, karena kelas kedua didefinisikan sebagai kelas pertama dengan akses ke "sertifikat" atau "bukti" ukuran polinomial.
Robin Kothari
ah benar karena BQP dan QMA keduanya memiliki elemen acak, tidak seperti BPP dan NP (lih: pertanyaan lain tentang hubungan antara QMA dan NP: cstheory.stackexchange.com/questions/1443/understanding-qma )
Suresh Venkat
12

Hanya satu hal untuk ditambahkan ke jawaban John:

Di bawah hipotesis derandomisasi yang masuk akal, AM = NP. Dalam hal ini, tentu kita akan memiliki AM ⊆ QMA.

Scott Aaronson
sumber